Đến nội dung

tranductucr1 nội dung

Có 195 mục bởi tranductucr1 (Tìm giới hạn từ 25-04-2020)



Sắp theo                Sắp xếp  

#715679 Nhờ mọi người bài tính ma trận A^n

Đã gửi bởi tranductucr1 on 17-09-2018 - 22:33 trong Đại số tuyến tính, Hình học giải tích

Ma trận A:

 

$\begin{pmatrix} 1 & -1 \\ 2 & 4 \end{pmatrix}$

tính A^n, với n=(1,2....)

p/s: mình học cùi bắp phần quy nạp lắm hix...

Dùng định lý Cayley-Hamilton ta có 
$|\lambda I_2-A|=|\begin{bmatrix} \lambda-1 & 1\\-2 & \lambda-4 \end{bmatrix}|=(\lambda-1)(\lambda-4)+2=\lambda^{2}-5\lambda+6$

=> $A^{n+2}-5A^{n+1}+6A^{n}=C$($C$ là ma trận không(1)
đặt $u_{(ij)(n)}$ là phần tử hàng  $i$ cột $j $ của ma trận $A^n$

Từ (1) ta có $u_{n+2}=5u_{n+1}-6u_{n}$ (Xét riêng cho trường hợp $i=j=1$, trường hợp còn lại tương tự)
Giải phương trình sai phân ta được $u_{n}=\alpha .2^{n}+\beta. 3^n$
từ đó tìm được $A^{n} $
Đáp số: $\begin{bmatrix} 2^{n+1}-3^{n} &2^{n}-3^{n} \\ -2^{n+1}+2.3^{n} & 5.2^{n}-2.3^{n} \end{bmatrix}$




#712882 Lớp có 45 học sinh trong đó có 17 nam, xếp thành một đường thẳng hỏi có bao n...

Đã gửi bởi tranductucr1 on 20-07-2018 - 16:04 trong Tổ hợp - Xác suất và thống kê - Số phức

D

 

Số phần tử không gian mẫu: $45!$

Số cách xếp bạn nam: $17!$

Số cách xếp bạn nữ: $28!$

Giữa các bạn nữ có $29$ chỗ trống, số cách xếp các bạn nam vào các chỗ trống này: $C_{29}^{1}$

XS cần tìm:

$P=\frac{C_{29}^{1}17!28!}{45!}$

Đọc kỹ đề đi  bạn




#711381 $2-10-14$

Đã gửi bởi tranductucr1 on 22-06-2018 - 08:40 trong Toán rời rạc

Một bài toán hay và hóc búa :

Một tiến sĩ độc ác Schrodinger đã phát minh tia phóng xạ kích thích tăng trưởng với mục đích tạo thành một binh đoàn mèo khổng lồ tấn công thành phố

attachicon.gifCapture.PNG

Để ngăn chặn ý đồ này, bạn cùng nhóm đặc vụ tiến vào căn phòng thí nghiệm của ông ta nhưng không may, bạn đã rơi vào bẫy, và căn phòng đổ sụp.  Schrodinger đã trốn sang một căn phòng khác và ngừng hệ thống. May sao, nhóm đặc vụ của bạn rất chuyên nghiệp và đặc vụ Delta đã khởi động lại hệ thống, trong khi đó, đặc vụ Alpha tìm kiếm mã mở khóa cho căn phòng tiếp theo và anh đã tìm được : $2-10-14$

attachicon.gifCapture2.PNG

Tất cả những điều mà bạn cần làm để thoát khỏi đó là nhập mã vào hệ thống. Nhưng có một vấn đề : hệ thống bị phá hỏng nên chỉ có 3 nút hoạt động : $+5$; $+7$ và $\sqrt{}$ và bạn phải tạo thành các số "2","10" và "14" theo thứ tự đó. Bạn có thể biểu diễn các số khác nhau nhưng không thể khởi động lại dãy số nên một khi bạn đạt được đến số "2" thì bạn phải tiếp tục biểu diễn "10" theo số "2" và "14" theo số "10". Tuy nhiên còn một vấn đề khác mà đặc vụ Delta giải thích cho bạn : Nếu có 2 số giống nhau hoặc một số $\geq60$ hoặc một số không tự nhiên xuất hiện trên màn hình hiển thị, căn phòng sẽ phát nổ. Hiện tại màn hình đang hiển thị số "0"

( Giả sử màn hình đang hiển thị số "19" ấn $+5$ sẽ được "24", ấn $+7$ được "31", ấn $+5$ được "36" và ấn $\sqrt{}$ được "6" )

attachicon.gifCapture3.PNG )

VẬY BẠN SẼ LÀM THẾ NÀO ĐỂ THOÁT KHỎI CĂN PHÒNG ?

Nguồn : Google  :D  

- Lao Hac and Korkot

p/s : Sorry mọi người em edit hơi tệ :D

 

Do có một số anh, chị chưa hiểu về đề nên em xin giải thích ạ :)

1. Có 2 số giống nhau 

Ví dụ sau khi biến đổi ra số "12" chẳng hạn. Tiếp tục biến đổi một hồi sau mà số "12" lại xuất hiện một lần nữa trên màn hình hiển thị thì phòng phát nổ

2. Có số $\geq60$

Đơn giản là nếu biến đổi ra số $\geq60$ thì phòng phát nổ . Ví dụ ấn 12 lần + 5 chẳng hạn

3. Có số " không tự nhiên"

Có nghĩa là biến đổi thành số thập phân ý ạ :) > Giả sử $\sqrt{12} = 3.464......$

3 TH đó sẽ khiến phòng phát nổ :)

LƯU Ý: CHỈ ĐƯỢC $+5$; $+7$ hoặc $\sqrt{}$

Mọi người khi giải chỉ cần ghi dãy số biến đổi, không cần ghi quy trình tạo nên dãy số đó :)

 

ĐƠN GIẢN HÓA BÀI TOÁN

Từ một số 0, chỉ được $+5, +7, \sqrt{}$ thì làm thế nào để tạo ra một dãy số trong đó có chứa các số "2" "10" và "14". Biết rằng 3 số này phải được sắp xếp theo thứ tự 2 -10 - 14, sao cho dãy số đó không có 2 số trùng lặp, không có số nào $>60$ và tất cả đều là các số tự nhiên

 

VẤN ĐỀ MỞ RỘNG :

Có bao nhiêu cách để tạo ra mật khẩu ?

$0,5,12,19,26,31,36,6,11,16,4,2,9,3,10,17,22,29,34,39,44,49,7,14 $




#701923 Bất đẳng thức và cực trị

Đã gửi bởi tranductucr1 on 20-02-2018 - 16:04 trong Bất đẳng thức và cực trị

$(x^2+y^2+z^2)(\frac{1}{x^2}+\frac{1}{y^2}+\frac{1}{z^2}) \geq (\frac{(x+y)^2}{2}+z^2)(\frac{8}{(x+y)^2}+\frac{1}{z^2}) =  \frac{(x+y)^2}{2z^2}+\frac{8z^2}{(x+y)^2}+5 \geq (\frac{(x+y)^2}{2z^2}+\frac{z^2}{2(x+y)^2})+\frac{15z^2}{2(x+y)^2}+5 \geq 6+\frac{15}{2} =\frac{27}{2}$




#690575 Tìm tham số $m$ để cắt đồ thị hàm số $y=\frac{x+m...

Đã gửi bởi tranductucr1 on 15-08-2017 - 10:58 trong Hàm số - Đạo hàm

Đường thẳng $y=2x+1$ cắt đồ thị hàm số $y=\frac{x+m}{x-1}$

$\Leftrightarrow$ phương trình $(2x+1)(x-1)=x+m$ có nghiệm khác $1$

$\Leftrightarrow$ phương trình $2x^2-2x-m-1=0$ có $2$ nghiệm phân biệt

$\Leftrightarrow \Delta '=1+2m+2=2m+3> 0\Leftrightarrow m> -\frac{3}{2}$

$m=\frac{-3}{2}$ vẫn đúng !!

*** Tại sao phần màu đỏ lại tương đương với nhau ? 




#690573 Tìm phép tịnh tiến biến $y=\dfrac{1}{x-2}$...

Đã gửi bởi tranductucr1 on 15-08-2017 - 10:44 trong Tổ hợp - Xác suất và thống kê - Số phức

Cho $(H)$: $y=\dfrac{1}{x-2}$

Tìm phép tịnh tiến biến $(H)$ thành $y=\dfrac{x-3}{x+7}$.

chỉ dùng phép tịnh tiến thôi thì sao biến được




#678479 Chứng minh rằng: $49\sum a^2+16\sum a^2b^2+24\sum ab...

Đã gửi bởi tranductucr1 on 24-04-2017 - 12:44 trong Bất đẳng thức - Cực trị

Đoạn nào không có ý nghĩa?

lời giải thiếu tự nhiên mang tính chất "máy tính"
lỡ may bạn đăng bài không biết làm thì lời giải này không giúp gì được cho bạn ấy cả 




#678178 Chứng minh rằng: $49\sum a^2+16\sum a^2b^2+24\sum ab...

Đã gửi bởi tranductucr1 on 21-04-2017 - 05:54 trong Bất đẳng thức - Cực trị

Trời nóng quá nên lười gõ TEX

 

20175ba99665-400f-4d6b-8891-1b70237aace5

Lời giải tuy ngắn gọn nhưng không có ý nghĩa gì nhỉ 




#677881 Đề thi HSG lớp 9 tỉnh Khánh Hòa 2016-2017

Đã gửi bởi tranductucr1 on 18-04-2017 - 17:02 trong Tài liệu - Đề thi

17953026_1886916034924948_14283146249250

Bài 6 . Một ví dụ kinh điển 
gọi $A,B,C,D,E,F$ là các điểm đại diện cho 6 người nói trên, 2 điểm bất kì nối với nhau bằng hai màu xanh hoặc đỏ (màu xanh chứng tỏ hai người đó quen nhau, màu đỏ là ngược lại). Ta đưa bài toán về chứng minh tồn tại tam giác có ba cạnh cùng màu 

  Ta có: xét các cạnh AB,AC,AD,AE,AF theo nguyên tắc Dirichlet: Tồn tại 3 cạnh cùng màu. Giả sử đó là màu xanh, và 3 cạnh đó là AB,AC,AD
Xét tam giác BCD
+Nếu 3 cạnh tam giác là màu đỏ ta có ngay dpcm
+Nếu trong 3 cạnh tam giác tồn tại ít nhất một cạnh màu xanh, giả sử đó là BC. Ta có ngay tam giác ABC thỏa mản điều kiện 
Tóm lại: Đa giác bất kì luôn có 1 tam giác "trùng màu"(Đpcm)




#677879 Đề thi HSG lớp 9 tỉnh Khánh Hòa 2016-2017

Đã gửi bởi tranductucr1 on 18-04-2017 - 16:52 trong Tài liệu - Đề thi

317953026_1886916034924948_14283146249250

3. từ đề ta có $\frac{1}{x}+\frac{1}{y}=\frac{4}{3}$
ta đó ta chỉ cần chứng minh $\frac{9}{2} \leq x^2+y^2 \leq 10 $ 
đặt $t=xy $ ta có $ t \geq 1$ => $ x^2+y^2=(x+y)^2-2xy=\frac{9t^2}{4}-2t$ 

Tiến hành khảo xát $f(t)=\frac{9t^2}{4} -2t$ với $t \geq \frac{9}{4}$  ta được dpcm




#675979 Tìm số tự nhiên $n$ thỏa mãn:

Đã gửi bởi tranductucr1 on 02-04-2017 - 10:17 trong Đa thức

Tìm số tự nhiên $n$ thỏa mãn: 

  $a^{n}(b-c)+b^{n}(c-a)+c^{n}(a-b)$ chia hết cho $a^{2}+b^{2}+c^{2}+ab+bc+ca$.

         với $a,b,c$ là các số thực bất kì.

nếu đặt $f(a,b,c)=a^{n}(b-c)+...$

ta thấy $f(a,b,c)$ sẽ có dạng  $f(a,b,c)=(a^{4}(b-c)+b^{4}(c-a)+c^{4}(c-a))g(a,b,c)$

đặt $g(a,b,c)=a^k+p(a,b,c)=b^k+q(a,b,c)=c^k+r(a,b,c)$
=>$f(a,b,c)=a^{4k}(b-c)+b^{4k}(c-a)+c^{4k}(a-b)+p(a,b,c)(b-c)+q(a,b,c)(c-a)+r(a,b,c)(a-b)$
để $f(a,b,c)$ có dạng $a^{n}(b-c)+b^{n}(c-a)+c^{n}(a-b)$ với mọi số thực $a,b,c$ thì 
$
p(a,b,c)(b-c)+q(a,b,c)(c-a)+r(a,b,c)(a-b) $ đồng nhất với 0

=> $p(a,b,c) \equiv q(a,b,c) \equiv r(a,b,c) \equiv 0$ 
=> $g(a,b,c)=a^k=b^k=c^k$ => k=0$
=> $g(a,b,c) \equiv 1$ 
$n=4$ là số duy nhất thỏa mãn đề toán 




#675820 $f(30)=4$ và $f(f(x))f(x)=79000$. Tính $f(1975)$

Đã gửi bởi tranductucr1 on 31-03-2017 - 18:24 trong Phương trình hàm

Theo đó ta cũng tồn tại số $a \in (4;30)$ sao cho $f(a)=30$ 

$\Rightarrow f(f(a)).f(a) =79000 \Leftrightarrow f(30).30=79000$ Từ đây ta có mâu thuẫn với đề bài

:v hay lắm bạn  :like  :like  :like  :like  :like  :like  :like  :like  :like  :like  :like  :like  :like  :like  :like




#675796 CMR $x^2 +y^4 +z^6 ≤ 2$

Đã gửi bởi tranductucr1 on 31-03-2017 - 12:24 trong Bất đẳng thức và cực trị

Trong $3$ số $x,y,z$ có ít nhất $2$ số cùng dấu. Giả sử $ x,y \geq 0$

$\rightarrow z=-x-y \leq 0$. Vì $-1 \geq x,y,z \geq 1$ $\rightarrow x^2+y^4+z^6 \geq |x| +|y|+|z| \geq 2$

Dấu $"="$ xảy ra khi $x=0;y=1;z=-1$

ý mình nói là với $|x|,|y|,|z|  \leq 1$ thì ta có $x^2+y^4+z^6 \leq |x|+|y|+|z|$ bài làm của bạn bị ngược dấu




#675720 Tìm số hạng tổng quát của dãy số un và tính tổng $S = \sum\lim...

Đã gửi bởi tranductucr1 on 30-03-2017 - 17:52 trong Đại số

Cho dãy số (un): $$\left\{ \begin{array}{l} {u_1} = \sqrt 2 \\ {u_{n + 1}} = \sqrt {4{u_n}^2 + 3} \end{array} \right.$$ với n>=1.

a) Tìm un

b) Tính $$S = \sum\limits_{i = 1}^{2016} {{u_i}^2} $$.

Đặt $y_n=u_n^2$
ta có $y_1=2$ và $y_{n+1}=4y_n+3$ 

=> $y_n=4^{n-1}y_1+3\frac{4^{n-1}-1}{4-1}=3*4^{n-1}-1$ 
=> $u_n=\sqrt{3*4^{n-1}-1}$ 

ta có $S=y_1+y_2+..+y_{2016}$ <=> $S=3(1+4+...+4^{2015})-2016=4^{2016}-2017$
 




#675707 $\lim_{x\rightarrow 1}\frac{x^{2017}-1}{x^{2000}-1}$

Đã gửi bởi tranductucr1 on 30-03-2017 - 13:38 trong Dãy số - Giới hạn

Tìm giới hạn: $\lim_{x\rightarrow 1}\frac{x^{2017}-1}{x^{2000}-1}.$

có 1 quy tắc mạnh để tính giới hạn dạng vô định

quy tắc này gọi là quy tắc L'Hopitan dùng này để thử lại kết quả thì rất nhanh 

phát biểu quy tắt như sau 

nếu $g(x_0)=0$ và $h(x_0)=0$ hàm số $g(x)$ và $h(x)$ có đạo hàm trên khoảng chưa $x_0$ 

ta luôn có $\lim_{x \rightarrow x_0}{\frac{g(x)}{h(x)}}=\lim_{x \rightarrow x_0}{\frac{g'(x)}{h'(x)}}$ (nếu tồn tại $\lim_{x \rightarrow x_0}{\frac{g'(x)}{h'(x)}}$)
Áp dụng bài trên ta có ngay $\lim_{x \rightarrow 1}{\frac{x^{2017}-1}{x^{2000}-1}}=\lim_{x \rightarrow 1}{\frac{2017x^{2016}}{2000x^{1999}}}=\frac{2017}{2000}$ 
từ đó ta có thể tổng quát lên 
$\lim_{x \rightarrow 1}{\frac{x^n-1}{x^m-1}}=\frac{n}{m}$




#675706 $\lim_{x\rightarrow 1}\frac{x^{2017}-1}{x^{2000}-1}$

Đã gửi bởi tranductucr1 on 30-03-2017 - 13:28 trong Dãy số - Giới hạn

Tìm giới hạn: $\lim_{x\rightarrow 1}\frac{x^{2017}-1}{x^{2000}-1}.$

$\lim_{x \rightarrow 1} {\frac{x^{2017}-1}{x^{2000}-1}} =\lim_{x \rightarrow 1}{\frac{x^{2016}+x^{2017}+..+1}{x^{1999}+x^{1998}+..+x+1}}=\frac{2017}{2000}$




#675704 Giới hạn một số hàm só mũ

Đã gửi bởi tranductucr1 on 30-03-2017 - 13:05 trong Dãy số - Giới hạn

a)$\lim_{x \rightarrow {+}\infty} ({\dfrac{1+x}{x-1}})^x$
b)$\lim_{x \rightarrow 0} \dfrac{(e)^{x^2}-cosx}{x^2}$ (e là số nepe)
c)$\lim_{x \rightarrow e} \dfrac{lnx-1}{x-e}$ (e là số nepe)
d)$\lim_{x \rightarrow{-}\infty} \dfrac{ln(1+3^x)}{ln(1+2^x)}$
e)$\lim_{x \rightarrow e} \dfrac{e^{-3x^2}-\sqrt[3]{1+x}}{ln(1+x^2)}$(e là số nepe)

Nếu được xin các bạn hướng dẫn đầy đủ. Cảm ơn.

$\lim_{x \rightarrow {+}\infty}({\dfrac{1+x}{x-1}})^x$

$=\lim_{x \rightarrow {+}\infty}{(1+\frac{2}{x-1})^x}$

$=\lim_{x \rightarrow {+}\infty}{(1+\frac{2}{x-1})^{\frac{x-1}{2}*\frac{2}{x-1}*x}}$

$=\lim_{x \rightarrow {+}\infty}{e^{\frac{2x}{x-1}}}$

$=e^2 $




#675703 CMR $x^2 +y^4 +z^6 ≤ 2$

Đã gửi bởi tranductucr1 on 30-03-2017 - 12:47 trong Bất đẳng thức và cực trị

Trong $3$ số $x,y,z$ có ít nhất $2$ số cùng dấu. Giả sử $ x,y \geq 0$

$\rightarrow z=-x-y \geq 0$. Vì $-1 \geq x,y,z \geq 1$ $\rightarrow x^2+y^4+z^6 \geq |x| +|y|+|z| \geq 2$

Dấu $"="$ xảy ra khi $x=0;y=1;z=-1$

bài làm  của bạn có nhiều mâu thuẫn chổ phần màu đỏ ? 

cách làm của tôi

ta có $x^2+y^4+z^6 \leq x^2+y^2+z^2=-2(xy+yz+xz) \leq 2$
vì ta luôn có $(1-x)(1-y)(1-z)+(x+1)(y+1)(z+1) \geq 0 \leftrightarrow xy+xz+yz \geq -1$




#671715 Cho $\frac{x}{a}+\frac{y}{b...

Đã gửi bởi tranductucr1 on 15-02-2017 - 19:36 trong Đại số

mình nghĩ đề là 

Cho $\frac{x}{a}+\frac{y}{b}+\frac{z}{c}=0$ và $\frac{a}{x}+\frac{b}{y}+\frac{c}{z}=2$. Tính giá trị:$A=\frac{a^{2}}{x^{2}}+\frac{b^{2}}{y^{2}}+\frac{c^{2}}{z^{2}}$

Ta có $$A=\frac{a^2}{x^2}+\frac{b^2}{y^2}+\frac{c^2}{z^2} =(\frac{a}{x}+\frac{b}{y}+\frac{c}{z})^2-2\frac{abz+bcx+acy}{xyz}=4-2\frac{abz+bcx+acy}{xyz}$$
Ta lại có $$\frac{x}{a}+\frac{y}{b}+\frac{z}{c}=0$$ 
=> $$abz+bcx+acy=0$$

Vậy $$A=4$$




#671524 tìm n $\in N$ biết $1+2^{2}+3^{2}+.....

Đã gửi bởi tranductucr1 on 13-02-2017 - 20:49 trong Số học

bạn có thể nói rõ hơn tại sao lại có công thức đó không?

 

C1 sử dụng quy nạp cái này đơn giản 
C2 ta có 
$S=1^2+2^2+...+n^2=1*(2-1)+2(3-2)+..+n(n+1-1)=1*2+2*3+3*4+...+n(n+1)-(1+2+3+4+...+n)$(3) 

ta có $1+2+3+...+n=\frac{n(n+1)}{2}$(2)
và $1*2+2*3+...+n(n+1)=\frac{1*2*3+2*3*3+...+n(n+1)*3}{3}=\frac{1*2*3+2*3*(4-1)+3*4*(5-2)+...+n(n+1)((n+2)-(n-1))}{3}=\frac{1*2*3-2*3*4+...+n(n+1)(n+2)-(n-1)n(n+1)}{3}=\frac{n(n+1)(n+2)}{3} $ (1)

thay (1) (2) vào (3) ta được 

$S=\frac{n(n+1)(n+2)}{3}-\frac{n(n+1)}{2}=\frac{n(n+1)(2n+1)}{6}$

Vừa nhớ ra một cách hay dùng để chứng minh tổng quát cho loại bài toán này :V 
 gọi $S_k=1^k+2^k+...+n^k$
ta có
$2^3=(1+1)^3=1^3+3*1^2+3*1+1^3$
$3^3=(1+1)^3=1^3+3*2^2+3*2+2^3$
...
$(n+1)^3=1^3+3*n^2+3*n+n^3$
=> $2^3+3^3+...+n^3+(n+1)^3=(1^3+1^3+...+1^3)+3(1^2+2^2+...+n^2)+3(1+2+..+n)+1^3+2^3+...+n^3$
=> $(n+1)^3=n+3S_2+3S_1+1$
=>$S_2=\frac{(n+1)^3-1-3S_1-n}{3}$
ta có kết quả quen thuộc $S_1=\frac{n(n+1)}{2}$
=>$S_2=\frac{n(n+1)(2n+1)}{6}$




#671517 tìm n $\in N$ biết $1+2^{2}+3^{2}+.....

Đã gửi bởi tranductucr1 on 13-02-2017 - 20:35 trong Số học

bạn có thể nói rõ hơn tại sao lại có công thức đó không?

C1 sử dụng quy nạp cái này đơn giản 
C2 ta có 
$S=1^2+2^2+...+n^2=1*(2-1)+2(3-2)+..+n(n+1-1)=1*2+2*3+3*4+...+n(n+1)-(1+2+3+4+...+n)$(3) 

ta có $1+2+3+...+n=\frac{n(n+1)}{2}$(2)
và $1*2+2*3+...+n(n+1)=\frac{1*2*3+2*3*3+...+n(n+1)*3}{3}=\frac{1*2*3+2*3*(4-1)+3*4*(5-2)+...+n(n+1)((n+2)-(n-1))}{3}=\frac{1*2*3-2*3*4+...+n(n+1)(n+2)-(n-1)n(n+1)}{3}=\frac{n(n+1)(n+2)}{3} $ (1)

thay (1) (2) vào (3) ta được 

$S=\frac{n(n+1)(n+2)}{3}-\frac{n(n+1)}{2}=\frac{n(n+1)(2n+1)}{6}$




#671073 $\frac{a}{a^2+1}+\frac{b}{b^2+1}+\frac{c}{c^2+1}\leq...

Đã gửi bởi tranductucr1 on 10-02-2017 - 23:22 trong Bất đẳng thức và cực trị

nếu $a,b,c\geq 0 ; a+b+c = 1 thì  M=\frac{a}{a^2+1}+\frac{b}{b^2+1}+\frac{c}{c^2+1}\leq \frac{9}{10}$

Ta có $a^{2}+\frac{1}{9}+\frac{1}{9}+...+\frac{1}{9} \geq 10\sqrt[10]{\frac{a^{2}}{9^9}}=10\frac{\sqrt[5]{a}}{9^{\frac{9}{10}}}$(Cosi 10 số)
=> $VT \leq \frac{9^{\frac{9}{10}}}{10}(a^{\frac{4}{5}}+b^{\frac{4}{5}} +c^{\frac{4}{5}})$
ta lại có => $\sqrt[5]{a^4} \leq \frac{\sqrt[5]{3}}{5}*( 4a+\frac{1}{3}) $(Cosi 5 số )
=> $VT \leq \frac{9^{\frac{9}{10}}}{10}(\frac{\sqrt[5]{3}}{5}(4(a+b+c)+1))$
=> $VT \leq \frac{9^{\frac{9}{10}}}{10}*\sqrt[5]{3} = \frac{9}{10}$ dpcm 




#671070 Cho hình chóp S.ABC có AB=AC, góc SAC= góc SAB. M là trung điểm BC.

Đã gửi bởi tranductucr1 on 10-02-2017 - 23:03 trong Hình học không gian

Cho hình chóp S.ABC có AB=AC, góc SAC= góc SAB. M là trung điểm BC. Chứng minh:

a. AM, SM vuông góc với BC

b. SA vuông góc với BC

(anh/chị nào giải được hướng dẫn e cách vẽ hình với ạ!)
 

ABC cân -> AM vuông góc BC 
ta có SBC cân (sử dụng định lý cosin cho hai tam giác SAB và SBC)
-> SM vuông góc với BC 
b) ta cần chứng minh $\underset{SA}{\rightarrow}*\underset{BC}{\rightarrow}=0$
$\Leftrightarrow \underset{SM}{\rightarrow}*\underset{BC}{\rightarrow}+\underset{MA}{\rightarrow}*\underset{BC}{\rightarrow}=0$ (đúng theo câu a)
 




#664058 $\boxed{Topic}$ÔN THI HỌC SINH GIỎI TOÁN 9 - NĂM HỌC...

Đã gửi bởi tranductucr1 on 07-12-2016 - 12:58 trong Chuyên đề toán THCS

Một bài bất đẳng thức, một bài phương trình. Mong mọi người tham gia đóng góp bài tập và giải bài

$\boxed{1}$ Ba số a, b, c thỏa mãn $0\leq a, b, c\leq 2$ và a + b + c = 3. Tìm giá trị lớn nhất của tổng A = a2 + b2 + c2.

$\boxed{2}$ Giải phương trình $\sqrt[3]{25x(2x^2+9)}=4x+\frac{3}{x}$

$(2-a)(2-b)(2-c)+abc \geq 0$ <=> $ab+bc+ac \geq 2 $

$A=(a+b+c)^2-2(ab+bc+ac) \leq 9-2*2=5$




#664055 Tìm min: $\frac{ab}{c}+\frac{bc}...

Đã gửi bởi tranductucr1 on 07-12-2016 - 12:48 trong Bất đẳng thức và cực trị

Cho a,b,c>0 thỏa mãn: $a^{2}+b^{2}+c^{2}=1$

Tìm GTNN của: $\frac{ab}{c}+\frac{bc}{a}+\frac{ca}{b}$

 áp dụng $ (a+b+c)^2 \geq 3(ab+bc+ac)$

ta có $(\frac{ab}{c}+\frac{bc}{a}+\frac{ac}{b})^2 \geq 3(a^2+b^2+c^2)$ ,,,,,